What is the solution to the inequality -6+|2p+3| > 7

Answers

Answer 1

Step-by-step explanation:

you're going to have to set up two expressions since it's an absolute value problem


Related Questions

Based on experience, the Ball Corporation’s aluminum can manufacturing facility in Ft. Atkinson, Wisconsin, knows that the metal thickness of incoming shipments has a mean of 0.2935 mm with a standard deviation of 0.000924 mm.

(a) A certain shipment has a diameter of 0.2963. Find the standardized z-score for this shipment.

Answers

Answer:

Step-by-step explanation:

the formula attached

PLz help!!

What is the degree of the polynomial

Answers

Answer:

3 maybe I'm just guessing.

now thats room for an answer again: yeah, 3 is right. its the x³ that defines the degree, its just the biggest power.

Find the y-intercept from the line passing through (1, 3) and having slope m=2.

Answers

Answer:

The y intercept is 1

Step-by-step explanation:

The slope intercept form of a line is

y = mx+b  where m is the slope and b is the y intercept

y = 2x+b

Substitute the point into the equation and solve for y

3 = 2(1)+b

3 =2+b

1 = b

The y intercept is 1

Y-intercept is (0,1)

Hope this helps! Please make me the brainliest, it’s not necessary but appreciated, I put a lot of effort and research into my answers. Have a good day, stay safe and stay healthy.

if one half of a number is 5 more than 6, what is the value when the number is tripled

Answers

Answer:

66

Step-by-step explanation:

let's use x to represent the unknown number

1/2x is 5 more than 6:

1/2x=5+6

solve to find x

1/2x=11

x=22

next, it asks us what is the value of the number when the number is tripled

since we already found what x is equal to, we can multiply that by 3 to figure out its value when it's tripled

3(22)=66

Using mathematical equation to model the scenario, the value of the number when tripled is 66

Let the number = n

0.5n = 5 + 6

0.5n = 11

Divide both sides by 0.5

n = 22

When n is tripled :

n = 22 × 3

n = 66

Hence, the value of the number when tripled is 66

Learn more : https://brainly.com/question/25480062

The length of a rectangle is 2 centimeters less than three times its width. Its area is 21 square centimeters. Find the dimensions of the rectangle. Use the​ formula, area=length*width.

Answers

Area = length x width

Area = 21 square cm

Width = x

Length = 3x + 2

21 = 3x+ 2 * x

21 = 3x ^2 + 2x

Subtract 21 from both sides:

3x^2 + 2x -21 = 0

Use the quadratic formula to solve for x:

-2 +/- sqrt(2^2-4*3(-21))/(2*3)

X = 7/3 and -3

A dimension can’t be a negative value so x needs to be 7/3

Width = x = 7/3 = 2 1/3 cm

Length = 3(7/3) + 2 = 9 cm

Check: 9 x 2 1/3 = 21

Dimensions: width 2 1/3 cm length 9 cm

Answer:

The dimensions of the rectangle are 7 by 3 centimeters.

Step-by-step explanation:

We are given that the length of a rectangle is two centimeters less than three times its width. In other words:

[tex]\displaystyle \ell = 3w-2[/tex]

Given that the area of the rectangle is 21 square centimeters, we want to determine the dimensions of the rectangle.

Recall that the area of a rectangle is given by:

[tex]A= w\ell[/tex]

Substitute:

[tex](21)=w(3w-2)[/tex]

Solve foro the width. Distribute:

[tex]3w^2-2w=21[/tex]

Isolate the equation:

[tex]3w^2-2w-21=0[/tex]

Factor. Find two numbers that multiply to 3(-21) = -63 and add to -2.

-9 and 7 suffice. Hence:

[tex]3w^2-9w+7w-21=0 \\ \\ 3w(w-3)+7(w-3) = 0 \\ \\ (3w+7)(w-3)=0[/tex]

Zero Product Property:

[tex]3w+7=0\text{ or } w-3=0[/tex]

Solve for each case. Hence:

[tex]\displaystyle w = -\frac{7}{3}\text{ or } w=3[/tex]

Since width cannot be negative, we can ignore the first solution.

Therefore, our width is three centimeters.

And since the length is two less than three times the width, the length is:

[tex]\ell = 3(3) - 2 = 7[/tex]

The dimensions of the rectangle are 7 by 3 centimeters.

Pls if anyone knows the answer that will be greatly appreciated :)

Answers

Answer:

I think the area is 60 but i couldn't figure out the perimeter, sorry.

Step-by-step explanation:

Answer:

perimeter = 36 m

area = 60 m²

Step-by-step explanation:

there is some missing information. for example about the types of the shapes. e.g. if the triangle on the top is an isosceles triangle (2 equal sides). or if the rectangle at the bottom is actually a square with 6 m on all sides. in order to make the sloped side of the top triangle a round, whole number, i assume that the bottom part is a square.

so, the area of this combined shape is the area of the bottom square plus the area of the top triangle.

area square As = 6×6 = 36 m²

so, one side of the triangle is also 6 m, the other is 14-6 = 8 m.

the area of such a right-angled triangle is half of the full rectangle of 6×8.

area triangle At = 6×8/2 = 48/2 = 24 m²

total area = As + At = 36 + 24 = 60 m²

the perimeter of the total shape is the sum of all sides.

so, 14, 6, 6 and ... the baseline/ Hypotenuse of the top triangle.

for that r need the mentioned Pythagoras :

c² = a² + b²

where a and b are the sides, and c is the Hypotenuse (the side opposite of the 90 degree angle).

so, in our case of an isosceles triangle with a 90 degree angle :

c² = 8² + 6² = 64 + 36 = 100

c = 10 m

so, the perimeter is

14+6+6+10 = 36 m

I really need help
Dz,2 of X is
(0-4)
(2,-2)
(6,2)

Answers

9514 1404 393

Answer:

  (6,2)

Step-by-step explanation:

As is often the case with multiple-choice problems, you don't actually need to know the detailed working. You just need to know what the answer looks like.

When point X is dilated by a factor of 2 with point Z as the center of dilation, it will move to a location twice as far from Z. You can tell by looking at the graph that X' will be in the first quadrant, above and to the right of the location of X. The only sensible answer choice is ...

  X' = (6, 2)

_____

Additional comment

X is a distance of X-Z = (4, 0) -(2, -2) = (2, 2) from Z Doubling that will put the image point a distance of 2(2, 2) = (4, 4) from Z. When this is added to Z, we find ...

  X' = Z + (4, 4) = (2+4, -2+4) = (6, 2)

Annie bought a 2 3/4 pound roast for the family dinner. A total of 9 people will be at dinner. How many pound of roast will each person get if the roast is divided up equally?

Answers

Answer:

11 /36 of a pound

Step-by-step explanation:

Take the pounds and divide by the number of people

2 3/4 ÷ 9

Change the mixed number to an improper fraction

(4*2+3)/4 ÷9

11/4 ÷9

Copy dot flip

11/4 * 1/9

11/36

Write expression with two terms that is equivalent to the expression shown. 4(2x + 11 - x)

Answers

Answer:

4x + 44

Step-by-step explanation:

4(2x + 11 - x) =

8x + 44 - 4x =

4x + 44

Need help! Thank you :)

Answers

Answer:

The value of [tex]x[/tex] is 10.

Step-by-step explanation:

Both angles ABC and CBD are complementary, that is, the sum of the measures of both angles equals 90°, that is:

[tex]\angle ABC + \angle CBD = 90^{\circ}[/tex] (1)

If we know that [tex]\angle ABC = 5\cdot x[/tex] and [tex]\angle CBD = 4\cdot x[/tex], then the value of [tex]x[/tex] is:

[tex]5\cdot x + 4\cdot x = 90^{\circ}[/tex]

[tex]9\cdot x = 90^{\circ}[/tex]

[tex]x = 10[/tex]

The value of [tex]x[/tex] is 10.

What is the difference between the temperature - 7 Celsius and - 12 Celsius on a scatter diagram

Answers

Answer:

Sự khác biệt giữa nhiệt độ - 7 độ C và - 12 độ C trên biểu đồ phân tán là gì

Step-by-step explanation:

What value of x makes the equation 3x +7=22

Answers

Answer:

x = 5

Step-by-step explanation:

Your goal is to isolate x

3x + 7 = 22

Subtract 7 from both sides and you are left with

3x = 15

In order to isolate x divide both sides by 3

x = 5

The pie chart shows student participation in fundraisers
at Mountain View Middle School.
Mountain View Middle School
Fundraisers
Which describes a bar diagram showing the percent of
students who participated in dances?
1 shaded square out of 10 squares
2 shaded squares out of 10 squares
3 shaded squares out of 10 squares
4 shaded squares out of 10 squares
20
Carnival
Car Wash
Bake Sale
Talent Show
10
50
5
15
Dances

Answers

If you add all of the numbers up, you get 100. Dances are colored purple, and that is 20. 20/100 simplified is 1/5. 1/5 is equivalent to 2/10. Therefore, the answer is 2 shaded squares out of 10 squares.

Answer:

Step-by-step explanation:

If you add all of the numbers up, you get 100. Dances are colored purple, and that is 20. 20/100 simplified is 1/5. 1/5 is equivalent to 2/10. Therefore, the answer is 2 shaded squares out of 10 squares.

Given the coordinates of two points on a line, explain two methods to determine the slope of the line.

Answers

Answer:

Step-by-step explanation:

1. Use the slope formula (y2 - y1 / x2 - x1)

2. Use the graph. Take two points and count the rise over the run.

Can you please help me

Answers

Answer:

you will type your answers and send

Answer:

Step-by-step explanation:

⅔-2/5 = 4/15

How many solutions will each system of linear equation have?

Answers

Answer:

The top system of equations has one solution, the middle system has infinitely many, and the bottom system has no solution.

Step-by-step explanation:

We can immediately see the top system has one solution because the two equations have different slopes.

For the middle system, we can rearrange terms and multiply by 3 to get that the equations are the same line, so there are infinitely many solutions.

Finally, we can move the -2x to the other side in the first equation of the bottom system to get 2x+y=5. But it also equals -7 from the second equation! This is impossible, so there are no solutions to the bottom system.

What is the value of the expression i 0 × i 1 × i 2 × i 3 × i 4?
1
–1
i
–i

Answers

Answer:

B. -1 should be the answer

Step-by-step explanation:

What is the maximum amount of a loan you can get if you pay $700 each month at a yearly rate of 0.89% for 10 years?

Answers

Answer:

$785.17

Step-by-step explanation:

Given data

PV is the loan amount

PMT is the monthly payment

i is the interest rate per month in decimal form (interest rate percentage divided by 12)

n is the number of months (term of the loan in months)

PMT =$700

n = 10 years

i = 0.89%

The formula for the loan amount is

Use the method of cylindrical shells to write out an integral formula for the volume of the solid generated by rotating the region bounded by the curve y = 2x - x^2 and the line y = x about the y-axis.

Answers

Answer:

The answer is "[tex]\frac{5\pi}{6}[/tex]"

Step-by-step explanation:

Please find the graph file.

[tex]h= y=2x-x^2\\\\r= x\\\\Area=2\pi\times r\times h\\\\= 2 \pi \times x \times (2x-x^2)\\\\= 2 \pi \times 2x^2-x^3\\\\volume \ V(x)=\int \ A(x)\ dx\\\\= \int^{x=1}_{x=0} 2\pi (2x^2-x^3)\ dx\\\\= 2\pi [(\frac{2x^3}{3}-\frac{x^4}{4})]^{1}_{0} \\\\= 2\pi [(\frac{2}{3}-\frac{1}{4})-(0-0)] \\\\= 2\pi \times \frac{5}{12}\\\\=\frac{5\pi}{6}\\\\[/tex]

In the figure, L1 || L2. 2x=174º. Find Za+Zb.

Answers

9514 1404 393

Answer:

  12°

Step-by-step explanation:

We assume you mean ...

  ∠x = 174°

Angle a is supplementary to angle x, so is ...

  ∠a = 180° -174° = 6°

Angle b is a vertical angle with respect to angle 'a', so is the same measure.

  ∠a +∠b = 6° +6° = 12°

I’m so confused. Need the help

Answers

1.621 kN

Step-by-step explanation:

Let the centerline of the canal be the x-axis. Because the forces exerted by the horses are symmetric to the centerline, only the x-components of these forces contribute to the resultant force on the barge, i.e., the y-components cancel out. Each x-component is equal to [tex]F_x = 839\cos 15[/tex] = 810.4 N. Therefore, the resultant force on the barge is twice this:

[tex]F_{net} = 2(839\:\text{N})\cos 15 = 1620.8\:\text{N}[/tex]

[tex]= 1.621\:\text{kN}[/tex]

Consider the sequence {an}={3n+13n−3n3n+1}. Graph this sequence and use your graph to help you answer the following questions.

Answers

Part 1: You can simplify [tex]a_n[/tex] to

[tex]\dfrac{3n+1}{3n}-\dfrac{3n}{3n+1} = \dfrac1{3n}+\dfrac1{3n+1}[/tex]

Presumably, the sequence starts at n = 1. It's easy to see that the sequence is strictly decreasing, since larger values of n make either fraction smaller.

(a) So, the sequence is bounded above by its first value,

[tex]|a_n| \le a_1 = \dfrac13+\dfrac14 = \boxed{\dfrac7{12}}[/tex]

(b) And because both fractions in [tex]a_n[/tex] converge to 0, while remaining positive for any natural number n, the sequence is bounded below by 0,

[tex]|a_n| \ge \boxed{0}[/tex]

(c) Finally, [tex]a_n[/tex] is bounded above and below, so it is a bounded sequence.

Part 2: Yes, [tex]a_n[/tex] is monotonic and strictly decreasing.

Part 3:

(a) I assume the choices are between convergent and divergent. Any monotonic and bounded sequence is convergent.

(b) Since [tex]a_n[/tex] is decreasing and bounded below by 0, its limit as n goes to infinity is 0.

Part 4:

(a) We have

[tex]\displaystyle \lim_{n\to\infty} \frac{10n^2+1}{n^2+n} = \lim_{n\to\infty}10+\frac1{n^2}}{1+\frac1n} = 10[/tex]

and the (-1)ⁿ makes this limit alternate between -10 and 10. So the sequence is bounded but clearly not monotonic, and hence divergent.

(b) Taking the limit gives

[tex]\displaystyle\lim_{n\to\infty}\frac{10n^3+1}{n^2+n} = \lim_{n\to\infty}\frac{10+\frac1{n^3}}{\frac1n+\frac1{n^2}} = \infty[/tex]

so the sequence is unbounded and divergent. It should also be easy to see or establish that the sequence is strictly increasing and thus monotonic.

For the next three, I'm guessing the options here are something to the effect of "does", "may", or "does not".

(c) may : the sequence in (a) demonstrates that a bounded sequence need not converge

(d) does not : a monotonic sequence has to be bounded in order to converge, otherwise it grows to ± infinity.

(e) does : this is true and is known as the monotone convergence theorem.

please answer all the questions and get 15 pts

Answers

Answer:

Here you go

Ans is in pictures.

Diane bought new headphones originally listed for $70.99. They are 25% off. Which equation can be used to find the amount Diane will save?

Answers

Step-by-step explanation:

100% = $70.99

there is a discount of 25%.

that means 75% (100 - 25) of the original price remains.

the equation to get any x% amount of a 100% total is simply

x% amount = 100% total amount × x/100

25% = 70.99 × 25/100 = $17.75

Now suppose that not every player can play in every position. The outfielders (left field, center field, right field) can play any outfield position, the infielders (1st base, 2nd base, 3rd base, short stop) can play any infield position, the pitchers can only pitch, and the catchers can only catch. Suppose a certain team has 20 players, of whom 3 are catchers, 4 are outfielders, 6 are infielders, and 7 are pitchers.
How many ways can the team assign field positions to 9 of the 19 players, putting each of the 9 selected players in a position he can play, and ensuring that all 9 field positions are filled?

Answers

Answer:

The team can assign field positions to 9 of the 19 players in 181,440 different ways.

Step-by-step explanation:

Since the outfielders (left field, center field, right field) can play any outfield position, the infielders (1st base, 2nd base, 3rd base, short stop) can play any infield position, the pitchers can only pitch, and the catchers can only catch, supposing a certain team has 20 players, of whom 3 are catchers, 4 are outfielders, 6 are infielders, and 7 are pitchers, to determine how many ways can the team assign field positions to 9 of the 19 players, putting each of the 9 selected players in a position he can play, and ensuring that all 9 field positions are filled, the following calculation must be performed:

3 x 7 x 6 x 5 x 4 x 3 x 4 x 3 x 2 = X

21 x 30 x 12 x 24 = X

630 x 12 x 24 = X

181,440 = X

Therefore, the team can assign field positions to 9 of the 19 players in 181,440 different ways.

Do 8 plz find OS thanks​

Answers

Answer:

OS = 10

Step-by-step explanation:

We can use a ratio to solve

QP     PR

-----  = ------

QO     OS

14          7

-----   = -------

14+6      OS

14          7

-----   = -------

20        OS

Using cross products

14 * OS = 7*20

14 OS = 140

Divide by 14

OS = 140/14

OS = 10

The answer above should be Cortes you

A kite is a quadrilateral with two pairs of adjacent, congruent sides. The vertex angles are those angles in between the pairs of congruent sides. Prove the diagonal connecting these vertex angles is perpendicular to the diagonal connecting the non-vertex angles. Be sure to create and name the appropriate geometric figures. This figure does not need to be submitted.​

Answers

Try the answer vertex

a family has two children. what is the probability that both are girls, given that at least one is a boy

Answers

Answer: The probability would be zero

Step-by-step explanation:

There are two children and one is a boy, so the probability of both being girls is a zero

Describe how to transform the graph of f(x) = x2 to obtain the graph of the related function g(x).
Then draw the graph of g(x).

1. g(x) = f(x + 1)
2. g(x) = f(x) - 2

Please help i also need to graph

Answers

9514 1404 393

Answer:

left 1 unitdown 2 units

Step-by-step explanation:

The transformation g(x) = f(x -h) +k is a translation of f(x) to the right by h units and up k units.

1. h = -1, so the graph of g(x) is the graph of f(x) shifted left 1 unit. (blue)

__

2. k = -2, so the graph of g(x) is the graph of f(x) shifted down 2 units. (green)

X^2 + bx + 49 is a perfect squad trinomial what is one possible value of b?

Answers

a perfect square trinomial, (x + y)² = x² + 2xy + y²

so, if we have the x of the bx, what is left is the b

the expression would have to be (x + 7)², since we have the 49 and the x²

so, what's left: x² + 14x + 49,

b = 14

hope it helps :)

Other Questions
Operating Leverage Beck Inc. and Bryant Inc. have the following operating data: Beck Inc. Bryant Inc. Sales $1,250,000 $2,000,000 Variable costs (750,000) (1,250,000) Contribution margin $500,000 $750,000 Fixed costs (400,000) (450,000) Operating income $100,000 $300,000 a. Compute the operating leverage for Beck Inc. and Bryant Inc. If required, round to one decimal place. Beck Inc. fill in the blank 1 Bryant Inc. fill in the blank 2 b. How much would operating income increase for each company if the sales of each increased by 20%? Dollars Percentage Beck Inc. $fill in the blank 3 fill in the blank 4 % Bryant Inc. $fill in the blank 5 fill in the blank 6 % c. The difference in the of operating income is due to the gi s bn l i trng ca khoa tham gia gii sinh vin khi nghip bn s p dng phong cach lnh o no? ti sao li chn phong cch Plz help guys thank u. What is the value of c in the interval (5,8) guaranteed by Rolle's Theorem for the function g(x)=7x3+91x2280x9? Note that g(5)=g(8)=9. (Do not include "c=" in your answer.) On a pie chart, a category representing 20% of the whole should correspond to a central angle of 20. An initially motionless test car is accelerated uniformly to 105 km/h in 8.43 s before striking a simulated deer. The car is in contact with the faux fawn for 0.635 s, after which the car is measured to be traveling at 60.0 km/h. What is the magnitude of the acceleration of the car before the collision?acceleration before collision:3.45m/s2 What is the magnitude of the average acceleration of the car during the collision?average acceleration during collision:19.68m/s2 What is the magnitude of the average acceleration of the car during the entire test, from when the car first begins moving until the collision is over? Edwin hiked to a famous point with a beautiful view. It took 2 hours and 55 minutes to hike to the viewpoint and 25 minutes to hike back. Edwin spent 25 minutes enjoying the view at the top. He finished the hike at 11:45 A.M. What time did Edwin start the hike to the viewpoint? Which of the following is the correct factorization of the polynomial below?x3 + 10x2 + 25xA. x(x + 5)(x - 5)B. (x2 + 2x - 5)(x-10)C. (x2 + 5x - 2)(x-10)D. x(x + 5)2 Please hurry this is due tomorrow morning A student is studying the ways different elements are similar to one another. Diagrams ofatoms from four different elements are shown below.Which two atoms are of elements in the same group in the periodic table?F Atom 1 and Atom 2G Atom 1 and Atom 4 H Atom 2 and Atom 3J Atom 3 and Atom 4 Describe what happens by the end of anaphase. a. The chromosomes in the cell become less tightly coiled. b. The nuclear envelope disappears in the cell. c. The mitotic spindle of the cell begins to form outside the nucleus. d. The cell begins to elongate and the two poles have an equivalent collection of chromsomes. e. The vesicles derived from the Golgi apparatus produce a cell plate at the middle of the cell that eventually forms the new cell wall. Please help me find the nouns and adjectives for this passage What is the value of the expression i 0 i 1 i 2 i 3 i 4?11ii Match the following vocabulary words The probability that Dan buys a sandwich is 0.2.The probability that Dan gets the bus is 0.9.Assuming the events are independent, what is the probability that Dan buys a sandwichand gets the bus? Zoe's Dog Biscuits, inc, has net cash flows from operating activities for the last year of $226 million. The income statement shows that net income in $150 million and depreciation expense is $85 million. During the year, the change in inventory on the balance sheet was an increase of $14 million, change in accrued wages and taxes was an increase of $15 million and change in accounts payable was an increase of $10 million. At the beginning of the year the balance of accounts receivable was $45 million. What was the end of year balance for accounts receivable? A. $20 million B. $25 million C. $45 million D. $65 million A glass block in air has critical angle of 49. What will happen to a ray of light coming through the glass when it is incident at and angle of 50 at the glass air boundary? Illustrate with a diagram What is the value of X? The range of cells in column D and rows 1 to 10 is represented as: 1:10 D1:D10 D:D Wind instruments like trumpets and saxophones work on the same principle as the "tube closed on one end" that we examined in our last experiment. What effect would it have on the pitch of a saxophone if you take it from inside your house (76 degrees F) to the outside on a cold day when the outside temperature is 45 degrees F? satellite does not need any energy to revolve around the earth why